Pharmacotherapy Test: Venous Thromboembolism (VTE)

Approved & Edited by ProProfs Editorial Team
The editorial team at ProProfs Quizzes consists of a select group of subject experts, trivia writers, and quiz masters who have authored over 10,000 quizzes taken by more than 100 million users. This team includes our in-house seasoned quiz moderators and subject matter experts. Our editorial experts, spread across the world, are rigorously trained using our comprehensive guidelines to ensure that you receive the highest quality quizzes.
Learn about Our Editorial Process
| By Caitlinb
C
Caitlinb
Community Contributor
Quizzes Created: 12 | Total Attempts: 9,354
Questions: 60 | Attempts: 568

SettingsSettingsSettings
Pharmacotherapy Test: Venous Thromboembolism (VTE) - Quiz

A condition in which a blood clot forms most often in the deep veins of the leg is known as Venous thromboembolism (VTE. This causes pain for the patient and can be corrected through different ways. Below is a pharmacotherapy test on Venous Thromboembolism. Give it a shot and get to refresh your memory on all that we have learnt do far.


Questions and Answers
  • 1. 

    Incidence of VTE almost doubles each decade after age __.

    Correct Answer(s)
    55
    Explanation
    As individuals age, their risk of developing venous thromboembolism (VTE) increases. The incidence of VTE almost doubles with each passing decade after the age of 55. This means that individuals in their 60s have a higher risk than those in their 50s, those in their 70s have a higher risk than those in their 60s, and so on. This trend highlights the importance of age as a significant risk factor for VTE and emphasizes the need for preventative measures and awareness among older individuals.

    Rate this question:

  • 2. 

    Hypercoagulation disorders can be hereditary or acquired. Genetic may be deficiencies in anticoagulant proteins and acquired may be things such as pregnancy or _____.

    Correct Answer(s)
    cancer
    Explanation
    Hypercoagulation disorders can be caused by both genetic and acquired factors. Genetic factors can include deficiencies in anticoagulant proteins, while acquired factors can be conditions such as pregnancy or cancer. Cancer is known to increase the risk of developing hypercoagulation disorders due to various mechanisms, including the release of procoagulant substances and the activation of the clotting cascade. Therefore, cancer is a valid answer to complete the sentence.

    Rate this question:

  • 3. 

    VTE risk factors include:  (Check all that apply.)

    • A.

      HRT meds

    • B.

      Acute medical illness

    • C.

      Immobility

    • D.

      Trauma

    • E.

      Pregnancy

    • F.

      Cancer

    Correct Answer(s)
    A. HRT meds
    B. Acute medical illness
    C. Immobility
    D. Trauma
    E. Pregnancy
    F. Cancer
    Explanation
    VTE stands for venous thromboembolism, which is a condition where blood clots form in the veins. The given answer includes various risk factors for VTE. Hormone replacement therapy (HRT) medications can increase the risk of blood clots. Acute medical illness, such as severe infections or heart attack, can also increase the risk. Immobility, such as being bedridden or sitting for long periods, can lead to stagnant blood flow and increase the risk of clots. Trauma, such as fractures or major surgeries, can cause blood clots to form. Pregnancy increases the risk due to hormonal changes and pressure on veins. Cancer can also increase the risk of blood clots.

    Rate this question:

  • 4. 

    Warfarin inhibits the practical synthesis of biologically active forms of the vitamin K-dependent clotting factors: __, __, __, and __, as well as the regulatory factors proteins C and S. (Check all that apply.)

    • A.

      II

    • B.

      IV

    • C.

      V

    • D.

      VII

    • E.

      VIII

    • F.

      IX

    • G.

      X

    • H.

      XI

    • I.

      XII

    Correct Answer(s)
    A. II
    D. VII
    F. IX
    G. X
    Explanation
    Warfarin inhibits the synthesis of clotting factors II, VII, IX, and X. These clotting factors are essential for the normal clotting process in the body. By inhibiting their synthesis, warfarin prevents the formation of blood clots. Additionally, warfarin also inhibits the synthesis of regulatory factors proteins C and S, which help to regulate the clotting process. Therefore, the correct answer is II, VII, IX, and X.

    Rate this question:

  • 5. 

    The severity of pain correlates with the size of the thrombus.

    • A.

      True

    • B.

      False

    Correct Answer
    B. False
    Explanation
    The severity of pain does not necessarily correlate with the size of the thrombus. Pain intensity can vary depending on various factors such as the location of the thrombus, the individual's pain tolerance, and other underlying conditions. Therefore, it is incorrect to say that the severity of pain always correlates with the size of the thrombus.

    Rate this question:

  • 6. 

    Which lab test tests for a fibrin degradation product?

    • A.

      Doppler

    • B.

      ESR

    • C.

      D-dimer

    • D.

      WBC

    Correct Answer
    C. D-dimer
    Explanation
    D-dimer is a lab test that is used to detect the presence of fibrin degradation products in the blood. Fibrin is a protein involved in blood clotting, and when it breaks down, it produces D-dimer fragments. Elevated levels of D-dimer can indicate the presence of a blood clot or thrombus. Therefore, D-dimer is the correct lab test to assess for fibrin degradation products.

    Rate this question:

  • 7. 

    A D-dimer can be used as a sole diagnostic test. A doppler is not necessary for diagnosis if the d-dimer is positive.

    • A.

      True

    • B.

      False

    Correct Answer
    B. False
    Explanation
    The statement is false because a D-dimer test alone is not sufficient for diagnosis. A positive D-dimer result indicates the presence of blood clotting, but it does not specify the location or cause of the clot. A Doppler ultrasound is often necessary to confirm the presence and location of a clot. Therefore, a positive D-dimer result should be followed by further diagnostic tests, such as a Doppler ultrasound, to accurately diagnose and determine the appropriate treatment.

    Rate this question:

  • 8. 

    What is the gold standard used to diagnose DVT?

    • A.

      Ultrasonography

    • B.

      D-dimer

    • C.

      Venography

    Correct Answer
    C. Venography
    Explanation
    Venography is considered the gold standard for diagnosing deep vein thrombosis (DVT). It involves injecting a contrast dye into the veins and then taking X-ray images to visualize the blood flow and identify any blockages or clots. This method provides a direct and accurate assessment of the veins, allowing for precise detection of DVT. While ultrasonography and d-dimer tests are also commonly used in diagnosing DVT, they may not always provide definitive results and may require further confirmation through venography.

    Rate this question:

  • 9. 

    Venography is an expensive, invasive test that carries the risk of anaphylaxis and nephrotoxicity.

    • A.

      True

    • B.

      False

    Correct Answer
    A. True
    Explanation
    Venography is a medical procedure that involves injecting a contrast dye into the veins to visualize blood flow. It is considered expensive because it requires specialized equipment and skilled professionals. Additionally, it is invasive as it involves inserting a catheter into the veins. This invasive nature of the test carries risks such as anaphylaxis, a severe allergic reaction to the contrast dye, and nephrotoxicity, which refers to damage to the kidneys caused by the contrast dye. Therefore, the statement that venography is an expensive, invasive test that carries the risk of anaphylaxis and nephrotoxicity is true.

    Rate this question:

  • 10. 

    Which of the following is true regarding ultrasonography? Select all that apply.

    • A.

      Non-invasive

    • B.

      Commonly used for DVT diagnosis

    • C.

      Not commonly used for DVT diagnosis

    • D.

      Cannot detect small blood clots in distal veins

    • E.

      Can detect small blood clots in distal veins

    • F.

      Invasive

    Correct Answer(s)
    A. Non-invasive
    B. Commonly used for DVT diagnosis
    D. Cannot detect small blood clots in distal veins
    Explanation
    Ultrasonography is a non-invasive imaging technique commonly used for the diagnosis of deep vein thrombosis (DVT). However, it cannot detect small blood clots in distal veins. Therefore, the correct answers are: Non-invasive, commonly used for DVT diagnosis, and cannot detect small blood clots in distal veins.

    Rate this question:

  • 11. 

    If a patient's assessment indicates DVT but a doppler does not show DVT, what is your next step?

    • A.

      Check for Homan's sign.

    • B.

      Check lab tests.

    • C.

      Do an invasive test (such as angiography or venography).

    • D.

      Nothing, if the doppler is negative there is no need to go further.

    Correct Answer
    C. Do an invasive test (such as angiography or venography).
    Explanation
    If a patient's assessment indicates DVT but a doppler does not show DVT, the next step would be to do an invasive test such as angiography or venography. This is because a negative doppler does not completely rule out the presence of DVT, as it may not always be able to detect the clot. An invasive test would provide a more accurate and conclusive diagnosis by directly visualizing the blood vessels and identifying any blockages or clots.

    Rate this question:

  • 12. 

    Heparin dissolves clots.

    • A.

      True

    • B.

      False

    Correct Answer
    B. False
    Explanation
    Heparin does not dissolve clots, but rather prevents the formation of new clots and helps existing clots from growing larger. It works as an anticoagulant by inhibiting the clotting factors in the blood. Therefore, the statement that "Heparin dissolves clots" is false.

    Rate this question:

  • 13. 

    Which of the following is true regarding UFH? (Check all that apply.)

    • A.

      SQ onset of action 1-2 hours.

    • B.

      Good bioavailability.

    • C.

      IV if need rapid anticoagulation.

    • D.

      Intra- and interpatient variability to response.

    • E.

      T ½ is dose dependent.

    • F.

      Treatment dose related to patients weight.

    • G.

      Use actual body weight if not obese.

    • H.

      The therapeutic range is 1.5 to 2.5x the control aPTT value.

    • I.

      Check aPTT baseline, 4 hours after starting UFH infusion, and 4 hours after each dose change.

    • J.

      Pregnancy category X.

    • K.

      Pregnancy category B.

    Correct Answer(s)
    A. SQ onset of action 1-2 hours.
    C. IV if need rapid anticoagulation.
    D. Intra- and interpatient variability to response.
    E. T ½ is dose dependent.
    F. Treatment dose related to patients weight.
    G. Use actual body weight if not obese.
    H. The therapeutic range is 1.5 to 2.5x the control aPTT value.
    Explanation
    UFH (unfractionated heparin) has a slow onset of action when administered subcutaneously (SQ), taking 1-2 hours to exert its anticoagulant effects. It can be administered intravenously (IV) if rapid anticoagulation is needed. There is intra- and interpatient variability in response to UFH, meaning that individuals may respond differently to the same dose. The half-life (T ½) of UFH is dose-dependent, meaning that higher doses result in a longer duration of action. The treatment dose of UFH is related to the patient's weight, and actual body weight should be used unless the patient is obese. The therapeutic range for UFH is 1.5 to 2.5 times the control activated partial thromboplastin time (aPTT) value.

    Rate this question:

  • 14. 

    Long term use of UFH can cause? (Check all that apply.)

    • A.

      Alopecia

    • B.

      Hyperkalemia

    • C.

      Osteoporosis

    Correct Answer(s)
    A. Alopecia
    B. Hyperkalemia
    C. Osteoporosis
    Explanation
    Long-term use of UFH (unfractionated heparin) can cause alopecia, hyperkalemia, and osteoporosis. Alopecia refers to hair loss, which can occur as a side effect of prolonged UFH use. Hyperkalemia is an elevated level of potassium in the blood, which can be caused by UFH use. Osteoporosis is a condition characterized by weakened bones, and long-term use of UFH can contribute to its development.

    Rate this question:

  • 15. 

    If platelets decrease, 50% from baseline or fall below 240,000, then suspect HIT.

    • A.

      True

    • B.

      False

    Correct Answer
    B. False
    Explanation
    if it falls below 120,000

    Rate this question:

  • 16. 

    If you suspect that a patient is excessively anticoagulated, what would your next step be?

    • A.

      Monitor Hgb, Hct, and BP. Lower the dose of UFH. Administer IV protamine sulfate.

    • B.

      Monitor Hgb, Hct, and BP. Discontinue UFH. Administer IV protamine sulfate.

    • C.

      Monitor Hgb, Hct, and BP. Lower the dose of UFH. Administer IV Vitamin K and fresh frozen plasma.

    • D.

      Monitor Hgb, Hct, and BP. Discontinue UFH. Administer IV Vitamin K and fresh frozen plasma.

    Correct Answer
    B. Monitor Hgb, Hct, and BP. Discontinue UFH. Administer IV protamine sulfate.
    Explanation
    If a patient is suspected to be excessively anticoagulated, the next step would be to monitor their hemoglobin (Hgb), hematocrit (Hct), and blood pressure (BP). Additionally, it is important to discontinue the use of unfractionated heparin (UFH) and administer intravenous (IV) protamine sulfate. Protamine sulfate is a medication that can reverse the effects of UFH by binding to it and neutralizing its anticoagulant activity. This helps to prevent further bleeding or complications associated with excessive anticoagulation.

    Rate this question:

  • 17. 

    A woman has just given birth and is breastfeeding. The doctor wants to know if you can provide her UFH or LMWH? You tell the doctor. (Select all that apply.)

    • A.

      Not UFH, because it is not safe during breastfeeding.

    • B.

      Not UFH, because there is a risk of maternal hemorrhage.

    • C.

      LMWH is a better alternative because it does not cross the placenta.

    • D.

      These drugs cross the placenta.

    Correct Answer(s)
    B. Not UFH, because there is a risk of maternal hemorrhage.
    C. LMWH is a better alternative because it does not cross the placenta.
    Explanation
    The correct answer is Not UFH, because there is a risk of maternal hemorrhage. UFH (unfractionated heparin) carries a higher risk of maternal hemorrhage compared to LMWH (low molecular weight heparin). Additionally, LMWH is a better alternative because it does not cross the placenta, which is important during breastfeeding to avoid any potential harm to the baby.

    Rate this question:

  • 18. 

    The doctor decides to give the woman an LMWH. What should we monitor?

    • A.

      APTT

    • B.

      Anti-factor Xa

    • C.

      Factor IIa

    • D.

      INR

    Correct Answer
    B. Anti-factor Xa
    Explanation
    LMWH stands for Low Molecular Weight Heparin, which is a type of anticoagulant medication. The doctor decides to give the woman LMWH, and when using LMWH, it is important to monitor the anti-factor Xa levels. This is because anti-factor Xa is a laboratory test that measures the activity of LMWH in the blood. Monitoring anti-factor Xa levels helps ensure that the LMWH is being administered at the correct dosage to prevent blood clotting or excessive bleeding. Therefore, in this case, monitoring anti-factor Xa levels is necessary to ensure the effectiveness and safety of the LMWH treatment.

    Rate this question:

  • 19. 

    LMWH has a longer half-life than UFH.

    • A.

      True

    • B.

      False

    Correct Answer
    A. True
    Explanation
    LMWH (Low Molecular Weight Heparin) has a longer half-life than UFH (Unfractionated Heparin). This means that LMWH stays in the body for a longer period of time compared to UFH. The longer half-life of LMWH allows for less frequent dosing, making it more convenient for patients. Additionally, the longer duration of action of LMWH provides more stable and predictable anticoagulation effects. UFH, on the other hand, has a shorter half-life and requires more frequent dosing to maintain therapeutic levels.

    Rate this question:

  • 20. 

    When would you give BID dosing when using an LMWH? Select all that apply.

    • A.

      Obesity

    • B.

      Pregnancy

    • C.

      < 50kg

    • D.

      Cancer

    • E.

      Renal failure

    Correct Answer(s)
    A. Obesity
    D. Cancer
    Explanation
    LMWH (Low Molecular Weight Heparin) is a type of anticoagulant medication used to prevent blood clots. BID dosing refers to administering the medication twice a day. In the given options, both obesity and cancer are conditions that increase the risk of blood clot formation. Therefore, BID dosing may be necessary to ensure adequate anticoagulation in these patients. However, pregnancy, being less than 50kg in weight, and renal failure do not necessarily indicate a need for BID dosing with LMWH.

    Rate this question:

  • 21. 

    When would you use anti-Xa analysis when using LMWH? Select all that apply.

    • A.

      In every patient

    • B.

      Renal impairment

    • C.

      Renal impairment

    • D.

      Weight < 50 kg

    • E.

      Morbid obesity (BMI > 50 or > 150kg)

    • F.

      Pregnancy

    • G.

      Adults

    • H.

      Children

    Correct Answer(s)
    C. Renal impairment
    D. Weight < 50 kg
    E. Morbid obesity (BMI > 50 or > 150kg)
    F. Pregnancy
    H. Children
    Explanation
    Anti-Xa analysis is used when using low molecular weight heparin (LMWH) in certain patient populations. It is used in patients with renal impairment because LMWH is primarily eliminated through the kidneys, and renal impairment can affect the clearance of the drug. Additionally, anti-Xa analysis is used in patients with weight less than 50 kg because dosing adjustments may be necessary in these patients. Morbidly obese patients with a BMI greater than 50 or weight greater than 150 kg may also require anti-Xa analysis as LMWH dosing may need to be adjusted. Pregnancy is another indication for anti-Xa analysis as LMWH may be used for anticoagulation during pregnancy. Lastly, anti-Xa analysis may be used in children to ensure appropriate dosing of LMWH.

    Rate this question:

  • 22. 

    If monitoring anti-Xa activity when a patient is on an LMWH, get sample __ hours after SC dose was given (peak concentration).

    Correct Answer(s)
    4
    Explanation
    The correct answer is 4 because monitoring anti-Xa activity is done to assess the peak concentration of LMWH in the patient's bloodstream. By obtaining a sample 4 hours after the subcutaneous dose, it allows for enough time for the drug to reach its peak concentration and provides an accurate measurement of its activity.

    Rate this question:

  • 23. 

    Prior to LMWH therapy, we should monitor? Select all that apply.

    • A.

      Serum creatinine

    • B.

      Baseline PT/INR

    • C.

      APTT

    • D.

      CBC with platelet

    Correct Answer(s)
    A. Serum creatinine
    B. Baseline PT/INR
    C. APTT
    D. CBC with platelet
    Explanation
    Prior to LMWH therapy, it is important to monitor several factors to ensure the safety and effectiveness of the treatment. The baseline PT/INR (prothrombin time/international normalized ratio) should be measured to assess the patient's clotting ability and to determine the appropriate dosage of LMWH. The aPTT (activated partial thromboplastin time) should also be monitored to assess the patient's intrinsic clotting pathway. Additionally, a CBC (complete blood count) with platelet count should be done to evaluate the patient's overall blood cell count and platelet function. Lastly, the serum creatinine level should be checked to assess kidney function, as LMWH is primarily eliminated through the kidneys.

    Rate this question:

  • 24. 

    You can use an LMWH if a patient has a history of HIT.

    • A.

      True

    • B.

      False

    Correct Answer
    B. False
    Explanation
    LMWH stands for Low Molecular Weight Heparin, which is a type of blood thinner commonly used to prevent and treat blood clots. HIT, or Heparin-Induced Thrombocytopenia, is a potentially life-threatening condition where the body develops an immune response to heparin, leading to a decrease in platelet count and an increased risk of blood clots. If a patient has a history of HIT, they should not be given LMWH as it is derived from heparin and can potentially trigger the same immune response. Therefore, the statement that "You can use an LMWH if a patient has a history of HIT" is false.

    Rate this question:

  • 25. 

    Guidelines recommend UFH if CrCl < 30 mL/min.

    • A.

      True

    • B.

      False

    Correct Answer
    A. True
    Explanation
    The given statement is true because according to the guidelines, Unfractionated Heparin (UFH) is recommended for patients with a Creatinine Clearance (CrCl) of less than 30 mL/min. This is because UFH is primarily cleared by the kidneys, and in patients with reduced kidney function, the clearance of UFH is decreased. Therefore, UFH is preferred in patients with lower CrCl values to avoid excessive accumulation and potential bleeding complications.

    Rate this question:

  • 26. 

    Protamine neutralizes 60% of LMWH activity. We should give a _____ protamine dose if the LMWH was given greater than 8 hours ago.

    • A.

      Larger

    • B.

      Smaller

    Correct Answer
    B. Smaller
    Explanation
    If protamine neutralizes 60% of LMWH activity, it means that it reduces the effectiveness of LMWH. Therefore, if the LMWH was given more than 8 hours ago, it would be appropriate to give a smaller dose of protamine to counteract the remaining LMWH activity. A smaller dose of protamine would be sufficient to neutralize the reduced amount of LMWH present in the body.

    Rate this question:

  • 27. 

    Both UFH and LMWH have IIa inhibitor activity. Arixtra has...

    • A.

      A little thrombin IIa inhibition

    • B.

      A lot of thrombin IIa inhibition

    • C.

      No thrombin IIa inhibition

    Correct Answer
    C. No thrombin IIa inhibition
    Explanation
    Arixtra does not have any thrombin IIa inhibition.

    Rate this question:

  • 28. 

    Arixtra (a factor Xa inhibitor) does not affect platelet function or aggregation.

    • A.

      True

    • B.

      False

    Correct Answer
    A. True
    Explanation
    Arixtra, as a factor Xa inhibitor, specifically targets and inhibits the activity of factor Xa, which is involved in the coagulation cascade. By inhibiting factor Xa, Arixtra prevents the formation of blood clots. However, it does not have any direct effect on platelet function or aggregation. Platelets play a crucial role in clot formation by sticking together and forming a plug at the site of injury. Since Arixtra does not interfere with platelet function or aggregation, it does not affect the normal clotting process mediated by platelets. Therefore, the statement "Arixtra does not affect platelet function or aggregation" is true.

    Rate this question:

  • 29. 

    Arixtra is indicated for:  (Select all that apply)

    • A.

      Prophylaxis of DVT

    • B.

      Treatment of acute DVT when given with warfarin

    • C.

      Treatment of PE when given with warfarin and therapy initiated in the hospital

    Correct Answer(s)
    A. Prophylaxis of DVT
    B. Treatment of acute DVT when given with warfarin
    C. Treatment of PE when given with warfarin and therapy initiated in the hospital
    Explanation
    Arixtra is indicated for the prophylaxis of deep vein thrombosis (DVT), which means it is used to prevent the formation of blood clots in patients who are at risk. It is also indicated for the treatment of acute DVT when given with warfarin, which suggests that Arixtra can be used in conjunction with warfarin to treat existing blood clots. Additionally, Arixtra is indicated for the treatment of pulmonary embolism (PE) when given with warfarin and therapy initiated in the hospital, indicating that it can be used to treat blood clots in the lungs when combined with warfarin and when the treatment is started in a hospital setting.

    Rate this question:

  • 30. 

    Which of the following is true regarding Arixtra?  Select all that apply.

    • A.

      Predictable and linear dose response

    • B.

      Rapid onset of action

    • C.

      Renally eliminated

    • D.

      Once daily administration (but anticoagulation continues for 2 to 4 days after discontinuation of the drug)

    • E.

      Long t ½ = 17 – 21 hours

    • F.

      No liver metabolism

    Correct Answer(s)
    A. Predictable and linear dose response
    B. Rapid onset of action
    C. Renally eliminated
    D. Once daily administration (but anticoagulation continues for 2 to 4 days after discontinuation of the drug)
    E. Long t ½ = 17 – 21 hours
    F. No liver metabolism
    Explanation
    Arixtra exhibits a predictable and linear dose response, meaning that the effect of the drug is directly proportional to the dose administered. It also has a rapid onset of action, providing quick anticoagulation effects. Arixtra is renally eliminated, meaning that it is primarily excreted through the kidneys. It is administered once daily, but its anticoagulation effects can continue for 2 to 4 days after the drug is discontinued. Arixtra has a long half-life of 17 to 21 hours, indicating that it remains in the body for an extended period of time. Lastly, Arixtra does not undergo liver metabolism, suggesting that it is not affected by liver function or hepatic enzyme activity.

    Rate this question:

  • 31. 

    Arixtra is contraindicated if severe renal impairment.

    • A.

      True

    • B.

      False

    Correct Answer
    A. True
    Explanation
    Arixtra is contraindicated in patients with severe renal impairment because it is primarily eliminated through the kidneys. Severe renal impairment can lead to a decrease in drug clearance and an increase in drug levels in the body, which can potentially result in toxicity. Therefore, it is important to avoid using Arixtra in patients with severe renal impairment to prevent any adverse effects.

    Rate this question:

  • 32. 

    If a patient has a non-life threatening bleed from Arixtra therapy, the patient should receive...

    • A.

      Factor VIIa

    • B.

      Fresh frozen plasma

    • C.

      Protamine

    Correct Answer
    B. Fresh frozen plasma
    Explanation
    Fresh frozen plasma is the correct answer because it contains clotting factors and can help to replenish the patient's blood clotting ability. Arixtra therapy is an anticoagulant medication, so if a patient experiences a non-life threatening bleed while on this therapy, fresh frozen plasma can be given to help restore the blood's ability to clot and stop the bleeding. Factor VIIa is a clotting factor concentrate that may be used in certain situations, but it is not the first-line treatment for a non-life threatening bleed from Arixtra therapy. Protamine is used to reverse the effects of heparin, not Arixtra.

    Rate this question:

  • 33. 

    Arixtra is contraindicated in which of following:

    • A.

      Severe renal insufficiency

    • B.

      Children

    • C.

      Weight < 50 kg

    • D.

      Pregnancy

    • E.

      Bacterial endocarditis

    • F.

      Thrombocytopenia

    Correct Answer(s)
    A. Severe renal insufficiency
    C. Weight < 50 kg
    E. Bacterial endocarditis
    F. Thrombocytopenia
    Explanation
    Arixtra is contraindicated in severe renal insufficiency because it is primarily eliminated through the kidneys, and impaired renal function can lead to accumulation of the drug and increased risk of bleeding. It is also contraindicated in patients with weight less than 50 kg because the recommended dosage is not established for this population. Arixtra should not be used in patients with bacterial endocarditis because it can increase the risk of bleeding complications. Additionally, it is contraindicated in patients with thrombocytopenia, as it can further decrease platelet count and increase the risk of bleeding.

    Rate this question:

  • 34. 

    Lovenox and Arixtra prophylactic dosing is weight-based.

    • A.

      True

    • B.

      False

    Correct Answer
    B. False
    Explanation
    The statement is false because Lovenox and Arixtra prophylactic dosing is not weight-based. These medications are actually dosed based on factors such as renal function, age, and indication. Weight-based dosing is commonly used for other medications, but not for Lovenox and Arixtra.

    Rate this question:

  • 35. 

    Arixtra is probably preferable to Lovenox for the prevention of venous thromboembolism.

    • A.

      True

    • B.

      False

    Correct Answer
    A. True
    Explanation
    Arixtra is likely to be a better choice than Lovenox for preventing venous thromboembolism. This could be due to several reasons such as Arixtra being more effective in preventing blood clots, having fewer side effects, or being easier to administer. However, without further information, it is difficult to provide a specific explanation for why Arixtra is preferable.

    Rate this question:

  • 36. 

    This drug is approved for patients with HIT and 40% of patients develop antibodies to it (leading to anaphylaxis).  You also need to adjust the dose in renal failure.

    • A.

      Fondaparinux

    • B.

      UFH

    • C.

      Lepirudin

    • D.

      Hirudin

    Correct Answer
    C. Lepirudin
    Explanation
    Lepirudin is the correct answer because it is a direct thrombin inhibitor that is approved for patients with heparin-induced thrombocytopenia (HIT). Approximately 40% of patients develop antibodies to heparin, which can lead to anaphylaxis. Lepirudin is an alternative anticoagulant that can be used in these patients. Additionally, the dose of lepirudin needs to be adjusted in patients with renal failure.

    Rate this question:

  • 37. 

    Bivalirudin is not indicated for DVT.

    • A.

      True

    • B.

      False

    Correct Answer
    A. True
    Explanation
    Bivalirudin is a medication that is primarily used as an anticoagulant during certain medical procedures, such as percutaneous coronary intervention (PCI). It works by inhibiting the activity of thrombin, an enzyme involved in blood clotting. While bivalirudin may have some anticoagulant effects, it is not indicated for the treatment of deep vein thrombosis (DVT). DVT is typically managed with other medications, such as anticoagulants like heparin or warfarin. Therefore, the statement that "Bivalirudin is not indicated for DVT" is true.

    Rate this question:

  • 38. 

    Which of the following are true regarding Argatroban?  Select all that apply.

    • A.

      Monitor by checking aPTT

    • B.

      Alters INR

    • C.

      Dosage adjustment needed in renal impairment

    • D.

      Dosage adjustment needed in liver impairment

    • E.

      Approved for prevention or treatment of thrombosis in pts with HIT

    Correct Answer(s)
    A. Monitor by checking aPTT
    B. Alters INR
    D. Dosage adjustment needed in liver impairment
    E. Approved for prevention or treatment of thrombosis in pts with HIT
    Explanation
    Argatroban is a medication that is used for the prevention or treatment of thrombosis in patients with heparin-induced thrombocytopenia (HIT). It is important to monitor the patient's activated partial thromboplastin time (aPTT) while on Argatroban therapy to ensure appropriate dosing. Argatroban can also alter the international normalized ratio (INR), which is a measure of blood clotting time. Dosage adjustment is necessary in patients with liver impairment, as the drug is metabolized in the liver. Therefore, the correct statements are: Monitor by checking aPTT, Alters INR, Dosage adjustment needed in liver impairment, and Approved for prevention or treatment of thrombosis in pts with HIT.

    Rate this question:

  • 39. 

    Measure INR daily when both Argatroban and Warfarin are given together and discontinue the _________ when the INR > 4.

    • A.

      Argatroban

    • B.

      Warfarin

    Correct Answer
    A. Argatroban
    Explanation
    When both Argatroban and Warfarin are given together, it is important to measure the INR (International Normalized Ratio) daily. INR is a measure of blood clotting time and is used to monitor the effectiveness of Warfarin. If the INR value exceeds 4, it indicates that the blood is too thin and there is an increased risk of bleeding. Therefore, in such cases, the medication Argatroban should be discontinued to prevent excessive anticoagulation.

    Rate this question:

  • 40. 

    There is no known antidote for DTIs.

    • A.

      True

    • B.

      False

    Correct Answer
    A. True
    Explanation
    The statement "There is no known antidote for DTIs" means that there is currently no known cure or treatment specifically designed to counteract the effects of DTIs (Direct Thrombin Inhibitors). This implies that if someone is affected by DTIs, there is no known medication or remedy that can be administered to reverse or neutralize the effects of these inhibitors. Therefore, the correct answer is True.

    Rate this question:

  • 41. 

    Which of the following is true regarding warfarin?

    • A.

      It is highly protein bound

    • B.

      You should start at a dose of 5mg or less if the patient has CHF, liver failure, is taking interacting meds, or is at a high risk of bleeding

    • C.

      Loading doses are sometimes necessary

    • D.

      Pregnancy category X

    • E.

      If VTE then rapid acting anticoagulant overlapped with warfarin for a minimum of 3 days

    • F.

      Adjust dose based on total weekly dose

    • G.

      Upon initiation, monitor INR every 2-3 days for 1st week

    • H.

      GI bleed is the most common type of bleed

    • I.

      ICH bleed is the most common type of bleed

    • J.

      ICH risk if INR >4

    • K.

      An overdose should be corrected with sub-q Vitamin K

    • L.

      Drugs that affect CYP2C9, CYP3A4, CYP1A2 will most likely alter INR

    • M.

      Pharmacodynamic drug interactions may not change the INR

    Correct Answer(s)
    A. It is highly protein bound
    B. You should start at a dose of 5mg or less if the patient has CHF, liver failure, is taking interacting meds, or is at a high risk of bleeding
    D. Pregnancy category X
    F. Adjust dose based on total weekly dose
    G. Upon initiation, monitor INR every 2-3 days for 1st week
    H. GI bleed is the most common type of bleed
    J. ICH risk if INR >4
    L. Drugs that affect CYP2C9, CYP3A4, CYP1A2 will most likely alter INR
    M. Pharmacodynamic drug interactions may not change the INR
    Explanation
    Warfarin is highly protein bound, meaning that it binds strongly to proteins in the blood and has a limited distribution outside of the vascular system. It is recommended to start at a dose of 5mg or less in patients with congestive heart failure, liver failure, taking interacting medications, or at a high risk of bleeding to minimize the risk of adverse effects. Loading doses may be necessary in certain situations to achieve therapeutic levels more quickly. Warfarin is classified as pregnancy category X, indicating that it is contraindicated in pregnancy due to the potential for fetal harm. The dose of warfarin should be adjusted based on the total weekly dose to maintain the desired INR (international normalized ratio). Monitoring of INR should be done every 2-3 days during the first week of initiation to ensure appropriate anticoagulation. GI bleed is the most common type of bleeding associated with warfarin therapy. There is an increased risk of intracranial hemorrhage (ICH) if the INR exceeds 4. Drugs that affect the activity of CYP2C9, CYP3A4, and CYP1A2 enzymes are likely to alter the metabolism of warfarin and can affect its anticoagulant effect. Pharmacodynamic drug interactions may not directly change the INR but can still impact the overall anticoagulant effect of warfarin.

    Rate this question:

  • 42. 

    A patient presents with an INR of 5.5 and no risk of bleeding.  What would be the preferred next step?

    • A.

      Hold 1 to 2 doses and give PO vitamin K and resume at a lower dose

    • B.

      Hold 1 to 2 doses and resume at a lower dose

    Correct Answer
    B. Hold 1 to 2 doses and resume at a lower dose
    Explanation
    The patient has an INR of 5.5, which indicates that their blood is not clotting properly. However, since there is no risk of bleeding mentioned, the preferred next step would be to hold 1 to 2 doses of the medication that is causing the high INR and resume at a lower dose. This approach allows for the medication to be temporarily stopped to allow the INR to decrease, while also ensuring that the patient continues to receive the necessary medication at a lower dose to maintain therapeutic levels.

    Rate this question:

  • 43. 

    Which of the following is true regarding warfarin induced skin necrosis?

    • A.

      Rare

    • B.

      A patient is at higher risk if they have a large loading dose

    • C.

      A patient is at higher risk if they have protein C or S deficiency

    • D.

      Can progress to necrotic gangrene

    • E.

      Not severe

    • F.

      Usually manifests in fatty areas, abdomen, buttocks, breasts

    • G.

      Presents as eggplant-colored skin or maculopapular rash

    • H.

      If a patient has warfarin induced skin necrosis or purple toe syndrome, warfarin should be discontinued

    Correct Answer(s)
    A. Rare
    B. A patient is at higher risk if they have a large loading dose
    C. A patient is at higher risk if they have protein C or S deficiency
    D. Can progress to necrotic gangrene
    F. Usually manifests in fatty areas, abdomen, buttocks, breasts
    G. Presents as eggplant-colored skin or maculopapular rash
    H. If a patient has warfarin induced skin necrosis or purple toe syndrome, warfarin should be discontinued
    Explanation
    Warfarin-induced skin necrosis is a rare condition that can occur in patients taking warfarin. It is more likely to occur in patients who have received a large loading dose of warfarin or who have a deficiency in protein C or S. The condition can progress to necrotic gangrene, which is a severe complication. It usually manifests in fatty areas of the body such as the abdomen, buttocks, and breasts, and presents as eggplant-colored skin or a maculopapular rash. If a patient develops warfarin-induced skin necrosis or purple toe syndrome, it is recommended to discontinue the use of warfarin.

    Rate this question:

  • 44. 

    Cranberry juice raises the INR.

    • A.

      True

    • B.

      False

    Correct Answer
    A. True
    Explanation
    Cranberry juice is known to interact with certain medications, including warfarin, which is a blood-thinning medication. Warfarin works by affecting the INR (International Normalized Ratio), which measures the time it takes for blood to clot. Cranberry juice can increase the effects of warfarin, leading to a higher INR. Therefore, the statement that cranberry juice raises the INR is true.

    Rate this question:

  • 45. 

    If a patient has her first episode of idiopathic VTE with or without hypercoagulable condition we should treat for...

    • A.

      3 months

    • B.

      6 months

    • C.

      12 months or longer if low bleeding risk

    Correct Answer
    C. 12 months or longer if low bleeding risk
    Explanation
    Patients who experience their first episode of idiopathic VTE (venous thromboembolism) with or without a hypercoagulable condition should be treated for 12 months or longer if they have a low bleeding risk. This extended treatment duration is recommended to prevent the recurrence of VTE. However, if the patient has a high bleeding risk, the duration of treatment may be shorter.

    Rate this question:

  • 46. 

    A patient presents to the ER with her first DVT.  She has factor V Leiden, is being treated for high blood pressure, cancer, and asthma.  We should treat her for at least ____ and consider longer term therapy if there is low bleeding risk.

    • A.

      6 months

    • B.

      12 months

    • C.

      18 months

    • D.

      24 months

    Correct Answer
    D. 24 months
    Explanation
    The patient presents with her first DVT and has multiple risk factors, including factor V Leiden, high blood pressure, cancer, and asthma. Given the presence of these risk factors, it is important to treat her for an extended period to prevent recurrence of DVT. The answer of 24 months suggests that longer-term therapy should be considered if there is a low bleeding risk. This duration allows for comprehensive treatment and monitoring to reduce the risk of future DVT episodes.

    Rate this question:

  • 47. 

    General contraindications to anticoagulant therapy includes...

    • A.

      Active bleeding

    • B.

      Hemorrhagic tendencies (hemophilia)

    • C.

      Severe liver dx

    • D.

      Severe thrombocytopenia

    • E.

      Malignant HTN

    • F.

      Inability to supervise or monitor therapy

    Correct Answer(s)
    A. Active bleeding
    B. Hemorrhagic tendencies (hemophilia)
    C. Severe liver dx
    D. Severe thrombocytopenia
    E. Malignant HTN
    F. Inability to supervise or monitor therapy
    Explanation
    General contraindications to anticoagulant therapy include active bleeding, hemorrhagic tendencies (hemophilia), severe liver disease, severe thrombocytopenia, malignant hypertension, and inability to supervise or monitor therapy. These conditions pose a risk for increased bleeding or complications during anticoagulant treatment. Active bleeding and hemorrhagic tendencies indicate a higher risk of excessive bleeding, while severe liver disease and severe thrombocytopenia can impair the body's ability to form blood clots. Malignant hypertension may increase the risk of bleeding as well. Inability to supervise or monitor therapy can lead to inadequate dosing or missed doses, which can be dangerous when using anticoagulants.

    Rate this question:

  • 48. 

    We can consider using an anti-thrombolytic in DVT if there is risk for limb grangrene, the patient has shock or ________.

    • A.

      Hypertension

    • B.

      Hypotension

    Correct Answer
    B. Hypotension
    Explanation
    If a patient with deep vein thrombosis (DVT) has hypotension, it suggests that they have low blood pressure. This can be a dangerous condition as it indicates that there is inadequate blood flow to the organs and tissues of the body. In such cases, using an anti-thrombolytic medication can be considered as it helps to dissolve blood clots and improve blood flow. By doing so, it can prevent further complications like limb gangrene, which is the death of tissue due to lack of blood supply.

    Rate this question:

  • 49. 

    Patients should take warfarin in the morning.

    • A.

      True

    • B.

      False

    Correct Answer
    B. False
    Explanation
    Taking warfarin in the morning is not necessary. Warfarin is an anticoagulant medication that is commonly prescribed to prevent blood clots. The timing of when to take warfarin can vary depending on individual factors and the specific instructions given by the healthcare provider. Some patients may be advised to take warfarin in the evening to ensure consistent levels of the medication in the bloodstream. Therefore, the statement that patients should take warfarin in the morning is false.

    Rate this question:

  • 50. 

    If a patient has had two VTEs how long should she be on warfarin?

    • A.

      3 months

    • B.

      6 months

    • C.

      12 months

    • D.

      18 months

    • E.

      24 months

    Correct Answer
    E. 24 months
    Explanation
    If a patient has had two VTEs (venous thromboembolism), it is recommended that she be on warfarin for 24 months. VTEs are blood clots that form in the veins, usually in the legs or lungs. Warfarin is an anticoagulant medication that helps prevent the formation of blood clots. The extended duration of 24 months is likely recommended to reduce the risk of recurrent VTEs and ensure adequate prevention.

    Rate this question:

Quiz Review Timeline +

Our quizzes are rigorously reviewed, monitored and continuously updated by our expert board to maintain accuracy, relevance, and timeliness.

  • Current Version
  • Mar 21, 2023
    Quiz Edited by
    ProProfs Editorial Team
  • Nov 10, 2010
    Quiz Created by
    Caitlinb

Related Topics

Back to Top Back to top
Advertisement